How Is Angular Momentum Calculated in Different Pivot Scenarios?

In summary: Always remember: The moment of inertia Icm is always with respect to the center of mass, from which there is a symmetric distribution of mass (at least for homogeneous rigid bodies). So whenever you need to calculate the moment of inertia with respect to another parallel axis you can use the parallel axis theorem.
  • #1
Mia
5
0

Homework Statement


A 2.0-m measuring stick of mass 0.175 kg is resting on a table. A mass of 0.500 kg is attached to the stick at a distance of 74.0 cm from the center. Both the stick and the table surface are frictionless. The stick rotates with an angular speed of 5.30 rad/s.

(a) If the stick is pivoted about an axis perpendicular to the table and passing through its center, what is the angular momentum of the system?

(b) If the stick is pivoted about an axis perpendicular to it and at the end that is furthest from the attached mass, and it rotates with the same angular speed as before, what is the angular momentum of the system?

Homework Equations


L=Iω
I=MR^2
I=(1/12)ML^2
I=(1/3)ML^2
[/B]

The Attempt at a Solution



Okay so for the first question I used the moment of inertia formula and treated the mass as a point mass.
(0.500*0.740^2)=0.27
Plus the moment of inertia for the stick(rod) since it passing through the center I used I=(1/12)ML^2
(1/12)(0.175*2^2)=0.058
Then I added and multiplied by the angular velocity
0.27+0.058=0.33*5.3=1.7 kg*m/s

Now this answer was right however when I did the second one I repeated the same process but used I= (1/3)ML^2 because it at the end of the rod.

So I did
(0.500*0.740^2)=0.27
Then
(1/3)(0.175*2^2)=0.23
Then I added and multiplied by the angular velocity
0.27+0.23=0.50*5.3=2.65 rounded to 2.7 kg*m/s

It is saying both 2.65 and 2.7 were wrong. Where did I go wrong?
[/B]
 
Physics news on Phys.org
  • #2
Mia said:
Now this answer was right however when I did the second one I repeated the same process but used I= (1/3)ML^2 because it at the end of the rod.
I think what you're missing here is the parallel axis theorem.
 
  • #3
(Ron)^2=-1 said:
I think what you're missing here is the parallel axis theorem.

Is that I=Icm +Md^2?
 
  • #4
Mia said:
Is that I=Icm +Md^2?
Yes it is. :smile:
 
  • #5
(Ron)^2=-1 said:
Yes it is. :smile:
Okay so I would add that to the moment of inertia I had found for the stick then multiply by the angular velocity?
 
  • #6
Mia said:
Okay so I would add that to the moment of inertia I had found for the stick then multiply by the angular velocity?
You need to find the total moment of inertia with respect to the new axis.
 
  • #7
(Ron)^2=-1 said:
You need to find the total moment of inertia with respect to the new axis.
Okay I'm getting a little confused.
So I=Icm +Md^2?
Would I use Icm = 1/12ML^2 for the center of mass?
thenMd^2
would d=1m since that would be the distance from the end of the rod to the center?
 
  • #8
Mia said:
Would I use Icm = 1/12ML^2 for the center of mass?
thenMd^2
would d=1m since that would be the distance from the end of the rod to the center?
Yes, you're right! D is the distance from the centre of mass axis (axis of symmetry).

Always remember: The moment of inertia Icm is always with respect to the centre of mass, from which there is a symmetric distribution of mass (at least for homogeneous rigid bodies). So whenever you need to calculate the moment of inertia with respect to another parallel axis you can use the parallel axis theorem.
 
Last edited:

Related to How Is Angular Momentum Calculated in Different Pivot Scenarios?

1. What is angular momentum?

Angular momentum is a physical quantity that describes the rotational motion of an object. It is a vector quantity that depends on an object's mass, velocity, and the distance of the mass from the axis of rotation.

2. How is angular momentum calculated?

Angular momentum is calculated by multiplying an object's moment of inertia (a measure of its resistance to rotation) by its angular velocity (the rate at which it rotates). The equation for angular momentum is L = Iω, where L is angular momentum, I is moment of inertia, and ω is angular velocity.

3. What is the relationship between angular momentum and torque?

Torque is the force that causes an object to rotate, while angular momentum is the measure of an object's rotational motion. The two are related through Newton's second law of motion, which states that the net torque on an object is equal to the rate of change of its angular momentum.

4. How is angular momentum conserved?

Angular momentum is conserved in a closed system, meaning that it remains constant as long as there are no external torques acting on the system. This is known as the law of conservation of angular momentum and is a fundamental principle in physics.

5. How is angular momentum used in real-world applications?

Angular momentum is crucial in understanding the motion of objects such as spinning tops, planets, and satellites. It is also used in fields such as engineering, astronomy, and robotics, where rotational motion plays a significant role in designing and controlling systems.

Similar threads

  • Introductory Physics Homework Help
Replies
10
Views
1K
  • Introductory Physics Homework Help
Replies
10
Views
435
  • Introductory Physics Homework Help
Replies
12
Views
2K
  • Introductory Physics Homework Help
Replies
3
Views
423
  • Introductory Physics Homework Help
10
Replies
335
Views
9K
  • Introductory Physics Homework Help
Replies
10
Views
1K
  • Introductory Physics Homework Help
2
Replies
67
Views
1K
  • Introductory Physics Homework Help
Replies
12
Views
2K
  • Introductory Physics Homework Help
2
Replies
62
Views
10K
  • Introductory Physics Homework Help
Replies
5
Views
375
Back
Top